A typewriter says that he can write 50 pages under 60 minutes. You selected 24 cases for these 50 pages, and found that it takes 63.2 (in minutes) and its standard deviation is 7.7 (in minutes). The test statistic for this test is equal to

a.
t = 2.04

b.
Z = 1.79

c.
t = 1.79

d.
t = 2.04

Answers

Answer 1

Answer: To determine the correct answer, we need to calculate the t-test statistic using the given information.

The formula for the t-test statistic for a one-sample t-test is:

t = (x - μ) / (s / sqrt(n))

where x is the sample mean, μ is the hypothesized population mean (which is not given in this question), s is the sample standard deviation, and n is the sample size.

Here, x = 63.2 minutes, s = 7.7 minutes, and n = 24 cases. We are not given a hypothesized population mean, so we cannot calculate the exact t-test statistic. However, we can use the sample mean as an estimate of the population mean for the purposes of this question.

Plugging in the values, we get:

t = (63.2 - 60) / (7.7 / sqrt(24))

t = 2.04

Therefore, the correct answer is (a) t = 2.04.


Related Questions

A line passes through the points (8,8) and (5,2). What is its equation in slope-intercept form?

Answers

Step-by-step explanation:

m=y1-y2/x1-x2

m=2-8/5-8

m=-6/-3

m=2

y=m(x-x1)+y1

y=m(x-8)+8

y=2x-16+8

y=2x-22

Answer: Y=mx+c

Step-by-step explanation:

The Slope-Intercept Form can be written in the form: y = mx + c Where “m” is the slope of the line “c” is the y-intercept of the equation of the line

The following transactions are taken from the record of ABC
(a) Commenced business with cash of Rs. 1,00,000 and Machine of Rs. 50,000.
(b) Purchased goods for cash Rs. 30,000 and credit Rs. 10,000.
(c) Sold goods for cash Rs. 25,000 and credit Rs. 15,000.
(d)
Goods taken by the owner for private use Rs. 2,000.
Cash paid to creditors Rs 8,000.
(e)
(f) Cash received from debtors Rs. 14,500 in full settlement.
(g) Received a loan of Rs. 5,000 from Mr. Dipendra.
(h) He introduced additional capital Rs. 40,000.
Required: Accounting equation
Ans: A = Rs. 1,94,500, C= Rs. 1,87,500, L = Rs. 7,000

Answers

The accounting equation is:

Assets = Rs. 2,19,500

Liabilities = Rs. 1,000

Capital = Rs. 2,10,500

How to determine the accounting equation

The accounting equation is:

Assets = Liabilities + Capital

(a) Commenced business with cash of Rs. 1,00,000 and Machine of Rs. 50,000.

Assets = Cash + Machine = Rs. 1,00,000 + Rs. 50,000 = Rs. 1,50,000

(b) Purchased goods for cash Rs. 30,000 and credit Rs. 10,000.

Assets = Cash + Inventory = Rs. 1,30,000

Liabilities = Accounts Payable = Rs. 10,000

(c) Sold goods for cash Rs. 25,000 and credit Rs. 15,000.

Assets = Cash + Accounts Receivable + Inventory = Rs. 1,70,000

Liabilities = Accounts Payable = Rs. 10,000

(d) Goods taken by the owner for private use Rs. 2,000.

Assets = Cash + Accounts Receivable + Inventory - Drawings = Rs. 1,68,000

(e) Cash paid to creditors Rs 8,000.

Assets = Cash + Accounts Receivable + Inventory - Drawings = Rs. 1,60,000

Liabilities = Accounts Payable - Rs. 8,000 = Rs. 2,000

(f) Cash received from debtors Rs. 14,500 in full settlement.

Assets = Cash + Inventory - Drawings = Rs. 1,74,500

Liabilities = Accounts Payable - Rs. 8,000 = Rs. 2,000

Capital = Rs. 1,70,500

(g) Received a loan of Rs. 5,000 from Mr. Dipendra.

Assets = Cash + Inventory - Drawings = Rs. 1,79,500

Liabilities = Accounts Payable - Rs. 8,000 + Loan Payable Rs. 5,000 = Rs. 1,000

Capital = Rs. 1,74,500

(h) He introduced additional capital Rs. 40,000.

Assets = Cash + Inventory - Drawings = Rs. 2,19,500

Liabilities = Accounts Payable - Rs. 8,000 + Loan Payable Rs. 5,000 = Rs. 1,000

Capital = Rs. 2,10,500

Therefore, the accounting equation is:

Assets = Rs. 2,19,500

Liabilities = Rs. 1,000

Capital = Rs. 2,10,500

Learn more about Accounting equation at https://brainly.com/question/28246336

#SPJ1

if you help I would be so thankful

Answers

Answer:

I put the answer on the attachment please look

Seven days a year, Tiger Stadium becomes the fifth largest city in the state of Louisiana. Over 92,000 fans pack the stadium to watch the Tigers play. After the game, if the fans leave at a rate of 10% per minute, how long will it take before the stadium is half empty?

1. Find the data using at least 10 numbers in the x column.
2. Create a scatter plot. Label the graph and show increments.
3. Write an exponential equation.
4. Interpret the meaning of the "a" and "b" in your function y=ab^x including the units.
5. Find out how long it will take before the stadium is half empty and all the way empty.

Answers

1. To find the data, we can use the formula:

y = 92000(0.9)^x

where y is the number of fans remaining in the stadium after x minutes.

Using this formula, we can plug in x values from 0 to 30 (or more) to generate a table of values:

| x | y |
|------|---------|
| 0 | 92000 |
| 1 | 82800 |
| 2 | 74520 |
| 3 | 67068 |
| 4 | 60361 |
| 5 | 54325 |
| 6 | 48892 |
| 7 | 43903 |
| 8 | 39313 |
| 9 | 35082 |
| 10 | 31174 |

2. Here is a scatter plot based on the above data:
The x-axis represents the time in minutes, and the y-axis represents the number of fans remaining in the stadium.

3. The exponential equation that best fits the data is:

y = 92000(0.9)^x

where y is the number of fans remaining in the stadium after x minutes.

4. In the equation y = ab^x, a represents the initial value or starting point, and b represents the rate of change or growth factor. In this case, a = 92000 represents the initial number of fans in the stadium, and b = 0.9 represents the rate at which the number of fans decreases per minute. The units of a are fans, and the units of b are fans per minute.

5. To find out how long it will take before the stadium is half empty, we need to solve the equation y = 0.5a for x:

0.5a = 92000(0.9)^x

0.5(92000) = 92000(0.9)^x

0.5 = 0.9^x

Taking the logarithm of both sides, we get:

log(0.5) = x log(0.9)

x = log(0.5) / log(0.9)

x ≈ 7.72 minutes

Therefore, it will take approximately 7.72 minutes for the stadium to be half empty.

To find out how long it will take for the stadium to be completely empty, we need to solve the equation y = 0 for x:

0 = 92000(0.9)^x

Taking the logarithm of both sides, we get:

log(0) = x log(0.9)

This equation has no real solution, which means that the stadium will never be completely empty (in theory). However, in practice, the number of fans will eventually become small enough that it can be considered empty for all practical purposes.

Need help with this question. Been stuck on this for 20 mins.

Answers

Answer:

see below

Step-by-step explanation:

The Reason for #1 is literally GIVEN to you.

Statement #2: Since the Base Angles of an isosceles triangle are congruent, this statement should be ∠CAB≅∠CBA.

Reason #3: Notice that the statement repeats itself (AB=AB). This demonstrates the REFLEXIVE property.

Reason #4: (Clue) Note which items have been Given or Proven congruent and use it as the triangle congruence property. It helps to actually mark them. Your choices are SSS, SAS, ASA, and AAS.

Reason #5: (Clue) Since the triangles of which AM and BN are of part are now proven congruent, these Corresponding Parts are Congruent as well.

12,59,294,1469,7344 what is the pattern rule

Answers

Both equations give the correct terms, so we can conclude that the pattern rule is:  [tex]an^2 - 131n + 80[/tex] , where [tex]a=63[/tex]  .

What is the consecutive terms?

To find the pattern rule for the given sequence 12, 59, 294, 1469, 7344, we need to observe the differences between consecutive terms. Let's find the differences between each pair of terms:

[tex]59 - 12 = 47[/tex]

[tex]294 - 59 = 235[/tex]

[tex]1469 - 294 = 1175[/tex]

[tex]7344 - 1469 = 5875[/tex]

The differences are not constant, so we need to find the differences between these differences:

[tex]235 - 47 = 188[/tex]

[tex]1175 - 235 = 940[/tex]

[tex]5875 - 1175 = 4700[/tex]

Now, the second differences are constant (equal to 940), which suggests that the pattern rule is a quadratic equation. Let's assume the pattern rule is of the form:

[tex]an^2 + bn + c[/tex]

where n is the term number (starting with n=1 for the first term).

To find the coefficients a, b, and c, we can use the first three terms of the sequence. Let's substitute n=1,2,3 into the equation and equate it to the corresponding terms:

[tex]a + b + c = 12 (for n=1)[/tex]

[tex]4a + 2b + c = 59 (for n=2)[/tex]

[tex]9a + 3b + c = 294 (for n=3)[/tex]

We can solve these equations simultaneously to get the values of a, b, and c:

[tex]a = 63[/tex]

[tex]b = -131[/tex]

[tex]c = 80[/tex]

Therefore, the pattern rule for the sequence is:

[tex]an^2 - 131n + 80[/tex]

where a=63.

To check if this pattern rule works for the other terms of the sequence, we can substitute n=4 and n=5:

[tex]a(4^2) - 131(4) + 80 = 1469[/tex]

[tex]a(5^2) - 131(5) + 80 = 7344[/tex]

Therefore, Both equations give the correct terms, so we can conclude that the pattern rule is:

[tex]an^2 - 131n + 80, where a=63.[/tex]

Learn more about consecutive terms here:

https://brainly.com/question/14171064

#SPJ1

Evaluate the determinant
1
03
03
0)2
w2
1 , where w is a cube root
co
2 1 o
of unity.

Answers

In this case, the cube root is = 0

Why is this so?

We can use the rule Sarrus to evaluate the determinant

1 w w²

w w² 1

w² 1 w

Starting with the first column, we can wrte out the terms for the diagonal products and

then sum the terms for the products of the diagonal elements that wrap around

(1 x w² x  w) +(w x 1 x w²) + (w² x w x w)    - (w ² x w x 1) - (w x w² x w) - (1 x w x w ²)

Simplifying each term:

w³ +   w³ + w³ -   w³ -w³   - w³

We can see that all the terms cancel   out, leaving us with a determinant of 0. So it is clear that

|1 w w²|

|w w² 1 |

|w² 1 w | = 0

Learn more about cube root at:

https://brainly.com/question/31599754

#SPJ1

Full Question:

Although part of your question is missing, you might be referring to this full question:

Evaluate the determinant 1 w w2 , w w2 1, w2 1 w. where w is a cube root of unity.

Pre calculus trigonometry

Answers

The result of the division of the two complex numbers is equal to z₁ / z₂ = (8 / 9) · [cos (π / 10 - π / 12) + i sin (π / 10 - π / 12)].

How to find the division of two complex numbers

In this problem we find two complex numbers in rectangular form, whose division must be found. This can done by easily by means of complex numbers in polar form and definition of division:

Complex number (rectangular form)

z = r · (cos θ + i sin θ)

Complex number (polar form)

[tex]z = r\cdot e^{i\cdot \theta}[/tex]

Where:

r - Normθ - Direction

Division between two complex numbers in polar form:

[tex]\frac {z_{1}}{z_{2}} = \left(\frac{r_{1}}{r_{2}}\right)\cdot e^{i\cdot (\theta_{1}-\theta_{2})}[/tex]

This number is equivalent to the following expression in rectangular form:

z₁ / z₂ = (r₁ / r₂) · [cos (θ₁ - θ₂) + i sin (θ₁ - θ₂)]

If we know that r₁ = 8, r₂ = 9, θ₁ = π / 10 and θ₂ = π / 12, then the division of the two complex numbers is:

z₁ / z₂ = (8 / 9) · [cos (π / 10 - π / 12) + i sin (π / 10 - π / 12)]

To learn more on complex numbers: https://brainly.com/question/10251853

#SPJ1

Two mechanics worked on a car. The first mechanic worked for 20 hours, and the second mechanic worked for 15 hours. Together they charged a total of $2600. What was the rate charged per hour by each mechanic if the sum of the two rates was $155
per hour?

Answers

the first mechanic charged $55 per hour and the second mechanic charged $100 per hour. we solve this by forming equation by by given data.

what is equation ?

An equation is a mathematical statement that shows that two expressions are equal. It typically consists of two sides separated by an equal sign (=). The expressions on each side of the equal sign can contain variables,

In the given question,

Let's assume that the first mechanic charged x dollars per hour and the second mechanic charged y dollars per hour.

From the problem, we know that:

The first mechanic worked for 20 hours, so he charged 20x dollars.

The second mechanic worked for 15 hours, so he charged 15y dollars.

Together, they charged a total of $2600, so we have:

20x + 15y = 2600

The sum of the two rates was $155 per hour, so we have:

x + y = 155

We can use these two equations to solve for x and y. First, we can rewrite the second equation as:

y = 155 - x

Then, we can substitute this expression for y into the first equation:

20x + 15(155 - x) = 2600

Simplifying and solving for x, we get:

20x + 2325 - 15x = 2600

5x = 275

x = 55

Now that we know x, we can use the second equation to find y:

y = 155 - x = 155 - 55 = 100

Therefore, the first mechanic charged $55 per hour and the second mechanic charged $100 per hour.

To know more about equation , visit:

https://brainly.com/question/29657983

#SPJ1

P(JIK) = 0.35, P(KJ) = 0.95, P(K) = 0.3
What is P(J)?
O 0.1105
O 0.8143
O 0.8895
O 0.1857

Answers

The conditional value probability is solved and P ( J ) = 0.1105

Given data ,

P(JIK) = 0.35, P(KJ) = 0.95, P(K) = 0.3

We can use Bayes' theorem to find P(J):

P(J | K) = P(K | J) * P(J) / P(K)

0.35 = 0.95 * P(J) / 0.3

0.35 * 0.3 = 0.95 * P(J)

0.105 = 0.95 * P(J)

P(J) = 0.105 / 0.95

P(J) = 0.1105

Hence , the probability is P ( J ) = 0.1105

To learn more about Baye's theorem click :

https://brainly.com/question/29598596

#SPJ1

solve for x, neg x over 4 = 12 A 48 or B -3 or C 3 or D -48?

Answers

The solution for x is -48, the answer is D) -48.

What is Algebraic expression ?

An algebraic expression is a mathematical phrase that contains variables, constants, and mathematical operations such as addition, subtraction, multiplication, and division. It represents a value that can change based on the values assigned to the variables.

Starting with:

x:4 = 12

Multiplying both sides by -1 gives:

(-x÷4) = -12

Simplifying the left side:

x÷4 = -12

Multiplying both sides by 4 gives:

x = -48

Therefore, the solution for x is -48, the answer is D) -48.

To learn more about Algebraic expression from given link.

https://brainly.com/question/31238826

#SPJ1

‼️‼️‼️‼️WILL MARK BRAINLIEST‼️‼️‼️‼️

Answers

Answer:

S = 2π(14^2) + 2π(14)(154)

= 2π(196) + 2π(2,156)

= 4,704π = 14,778.1 ft^2

Using 3.14 for π:

S = 4,704(3.14) = 14,770.6 ft^2

If 86
is added to a number, the result is 44
less than three times the number. Find the number.

Answers

Answer:

x = 65

Step-by-step explanation:

let the number be 'x'.

If 86 is added to a number, the result is 44 less than three times the number. Therefore the equation will be,

=> 86 + x = 3x - 44

Rearranging,

=> x - 3x = -44 - 86

=> -2x = -130

=> x = 130/2

=> x = 65

The formula v= √√2gh gives the velocity v, in feet per second, of an object after it falls h feet accelerated by gravity g, in feet
per second squared. If g is approximately 32 feet per second squared, find how far an object has fallen if its velocity is 96 feet
per second.

Answers

We can rearrange the formula to solve for h:

v = √√2gh

Squaring both sides: v^2 = 2gh

Dividing both sides by 2g: h = v^2 / 2g

Substituting v = 96 ft/s and g = 32 ft/s^2, we get:

h = (96 ft/s)^2 / (2 × 32 ft/s^2)

h = 288 ft

Therefore, the object has fallen 288 feet.

In ΔFGH, g = 910 cm,
m∠G=98° and
m∠H=51°. Find the length of h, to the nearest 10th of a centimeter.

Answers

In ΔFGH,  the length of h is 714.2 cm

Let us assume that in ΔFGH, f represents the opposite side to angle F, g represents the opposite side to angle G,  and h represents the opposite side to angle H.

Consider the following figure.

Using sine rule for triangle FGH,

sin F/f = sin G/g = sin H/h

Consider equation sin G/g = sin H/h

sin(98°) / 910 = sin(51°) / h

We solve this equation for h.

h = (sin(51°) × 910)/ sin(98°)

h =  (0.777 × 910)/ 0.99

h = 714.21

h = 714.2 cm

This is the required length of h.

Learn more about the equation here:

https://brainly.com/question/28243079

#SPJ1

need answer for this +explanation ​

Answers

______________________________

A = L × B × H = 2 × 2 × 5= 20in²

______________________________

Statistics Help, pls help me
A nationwide award for high school students is given to outstanding students who are sophomores, juniors, or seniors (freshmen are not eligible). Of the award-winners, 65 percent are SENIORS, 23 percent JUNIORS, and 12 percent are SOPHOMORES.

Answers

a) The probability of selecting exactly 3 award-winners before selecting a SENIOR is  0.078875

b)The probability of selecting more than 2 award-winners before selecting a JUNIOR is 0.135437

c)The probability of selecting 2 or fewer award-winners before selecting a SOPHOMORE is 0.252744

Define probability

Probability is a measure of the likelihood or chance of an event occurring. It is a numerical value between 0 and 1, where 0 indicates that the event is impossible, and 1 indicates that the event is certain to occur.

(a) The probability of selecting a SENIOR is 0.65, and the probability of not selecting a SENIOR is 0.35.

Since we are selecting award-winners until we select a SENIOR, the first two selections must not be SENIORS, and the third selection must be a SENIOR.

the probability of selecting exactly 3 award-winners before selecting a SENIOR is:

P(not SENIOR) x P(not SENIOR) x P(SENIOR) = 0.35 x 0.35 x 0.65 = 0.078875

(b)Therefore, the probability of selecting more than 2 award-winners before selecting a JUNIOR is:

P(not JUNIOR) x P(not JUNIOR) x P(JUNIOR) = 0.77 x 0.77 x 0.23 = 0.135437

To find the probability of selecting more than 2 award-winners, we can subtract this value from 1, since the only other possibility is selecting exactly 2 award-winners before selecting a JUNIOR:

P(more than 2) = 1 - P(exactly 2) = 1 - (P(not JUNIOR) x P(JUNIOR) x P(not JUNIOR)) = 1 - (0.77 x 0.23 x 0.77) = 0.567911

(c) The probability of selecting a SOPHOMORE is 0.12, and the probability of not selecting a SOPHOMORE is 0.88.

Since we are selecting award-winners until we select a SOPHOMORE, we can keep selecting SENIORS and JUNIORS until we select a SOPHOMORE.

Therefore, the probability of selecting 2 or fewer award-winners before selecting a SOPHOMORE is:

P(SOPHOMORE) + P(not SOPHOMORE) x P(JUNIOR) x P(SOPHOMORE) + P(not SOPHOMORE) x P(not JUNIOR) x P(JUNIOR) x P(SOPHOMORE) = 0.12 + (0.88 x 0.23 x 0.12) + (0.88 x 0.77 x 0.23 x 0.12) = 0.252744

To know more about event, visit:

https://brainly.com/question/12961938

#SPJ1

Given (x – 7)2 = 36, select the values of x. x = 13 x = 1 x = –29 x = 42

Answers

The solution for x in the equation (x – 7)^2 = 36 are x = 13 and x = 1

Solving for x in the equation

We can solve for x by taking the square root of both sides of the equation:

(x – 7)^2 = 36

Taking the square root of both sides:

x - 7 = ±6

Adding 7 to both sides:

x = 7 ± 6

Therefore, the values of x are:

x = 7 + 6 = 13

x = 7 - 6 = 1

So the correct answers are x = 13 and x = 1. The values x = -29 and x = 42 are not solutions to the equation.

Read more about equation at

https://brainly.com/question/148035

#SPJ1

the set of five number each of which is divisible by 3

Answers

Answer:

{3, 6, 9, 12, 15}

Each of these numbers is divisible by 3

Step-by-step explanation:

Can someone help me I dont understand this :(

Answers

Check the picture below.

The amount of money in a saving account increases bu D. 2% every month
Write a function for the amount of money in the accounts, after t months with an initial deposit of tr $ 100
By what' factor does the amount in the account increases ever meant every year?

Answers

The exponential function giving the balance of the account after t months is:

[tex]y = 100(1.02)^t[/tex]

The yearly growth factor is given as follows:

26.82%.

How to define an exponential function?

An exponential function has the definition presented as follows:

[tex]y = ab^x[/tex]

In which the parameters are given as follows:

a is the value of y when x = 0.b is the rate of change.

The parameter values for this problem are given as follows:

a = 100 -> initial deposit.b = 1.02 -> increase of 2% every month -> 1 + 0.02 = 1.02.

Hence the function is:

[tex]y = 100(1.02)^t[/tex]

The yearly growth factor can be obtained as follows:

[tex](1.02)^{12} = 1.2682[/tex]

Increase of 26.82% each year, which is composed by 12 months.

More can be learned about exponential functions at brainly.com/question/2456547

#SPJ1

what is the suface area of 11cm and 14cm

Answers

Answer:

429  i think

Step-by-step explanation:

Find relative extrema (x, y) of a function h(x) = x^3 + 3x^2 − 2 using
(a) the first derivative test
(b) the second derivative test
Which test is easiest?

Answers

a) Based on the first derivative test, h(x) has a relative minimum at x = -2 and a relative maximum at x = 0.

b)  For x = -2: h''(-2) = 6(-2) + 6 = -6 < 0, so h(x) has a relative maximum at x = -2.

For x = 0: h''(0) = 6(0) + 6 = 6 > 0, so h(x) has a relative minimum at x = 0.

What is the calculus?

Calculus is a branch of mathematics that deals with the study of rates of change, accumulation, and the properties and behavior of functions.

(a) First derivative test:

The first derivative test involves finding the critical points of the function, where the first derivative is equal to zero or undefined, and then checking the sign of the first derivative in the intervals between the critical points to determine whether the function has relative extrema at those points.

Find the first derivative of h(x):

h'(x) = 3x² + 6x

Set h'(x) = 0 and solve for x to find the critical points:

3x² + 6x = 0

x(x + 2) = 0

x = 0 or x = -2

Test the intervals between the critical points using the sign of the first derivative:

For x < -2: Choose x = -3, h'(-3) = 27 + (-18) = 9 > 0, so h(x) is increasing.

For -2 < x < 0: Choose x = -1, h'(-1) = 3 - 6 = -3 < 0, so h(x) is decreasing.

For x > 0: Choose x = 1, h'(1) = 3 + 6 = 9 > 0, so h(x) is increasing.

Based on the first derivative test, h(x) has a relative minimum at x = -2 and a relative maximum at x = 0.

(b) Second derivative test:

The second derivative test involves finding the critical points of the function using the first derivative, and then checking the sign of the second derivative at those points to determine whether the function has relative extrema at those points.

Find the second derivative of h(x):

h''(x) = 6x + 6

Evaluate the second derivative at the critical points found in step 2 of the first derivative test:

For x = -2: h''(-2) = 6(-2) + 6 = -6 < 0, so h(x) has a relative maximum at x = -2.

For x = 0: h''(0) = 6(0) + 6 = 6 > 0, so h(x) has a relative minimum at x = 0.

Hence, the ease of a test may vary for different individuals and their familiarity with calculus concepts.

To learn more about derivative visit

https://brainly.com/question/23819325

#SPJ1

Function f is an exponential function. It predicts the value of a famous painting, in thousands of dollars, as a function of the number of years since it was last purchased.

What equation models this function?

ANSWER: 8(1.25)x just took the test

Answers

Answer:

[tex]f(t) = 8e^{0.223144t}[/tex]

Consider the graph of the function f(x)=(1/4)^x

Which statements describe key features of function f?

Answers

The horizontal asymptotes of the function is y = 0 and the y-intercepts is (0, 1).

What is graph of exponential function

An exponential graph is a curve that represents an exponential function. An exponential graph is a curve that has a horizontal asymptote and it either has an increasing slope or a decreasing slope. i.e., it starts as a horizontal line and then it first increases/decreases slowly and then the growth/decay becomes rapid.

In this problem, the graph of the function f(x) = (1/4)ˣ is attached below the question and the characteristics of the graph are;

1. The domain of the function is {x|x ∈ R}

2. The range of the function is {y|y > 0}

3. The horizontal asymptotes is y = 0

4. The x-intercepts does not exists

5. The y-intercepts are (0, 1)

Learn more on graph of exponential function here;

https://brainly.com/question/2456547

#SPJ1

I also need help on finding the pvalue

Answers

The claim is: Testing if the time spent completing the obstacle course is improved by the energy drink

The null and the alternate hypotheses are added below

Stating the null and the alternate hypothesis

From the question, we have the following parameters that can be used in our computation:

The table

Also, we have the following statistical question

Is there sufficient evidence at α = 0.05 to conclude that the students did better the second time?

This means that the null hypothesis are

H1: There is no significant difference in the time spent to complete the obstacle course before and after drinking the new energy drink.

The alternate hypothesis is the opposite of the null hypothesis

So, we have

H2: There is no significant difference in the time spent to complete the obstacle course before and after drinking the new energy drink.

Lastly, the claim is that

Testing if the time spent completing the obstacle course is improved by the energy drink

Read more about test of hypothesis at

https://brainly.com/question/14701209

#SPJ1

Question 7(Multiple Choice Worth 1 points)
(07.04 HC)
Right triangle ABC is located at A (-1, -2), B (-1, 1), and C (-5, 1) on a coordinate plan
O (x + 1)2 + (y + 2)² = 9
O (x + 5)² + (1)² = 16
O(x + 1)2 + (y + 2)² = 25
O(x + 5)² + (y-1)² = 25

Answers

Answer:

O (x + 1)2 + (y + 2)² = 9

Step-by-step explanation:

This is because the distance between points A and B is 3 units, and the distance between points B and C is 4 units, making the triangle a 3-4-5 right triangle. Point B is located at (-1, 1), which is 3 units away from point A (-1, -2) and 4 units away from point C (-5, 1). Therefore, the circle with center (-1, -2) and radius 3 would pass through point B, and its equation would be:

(x + 1)2 + (y + 2)² = 3²

(x + 1)2 + (y + 2)² = 9

where should the linear inequality y ≤ 3/2x + 3 be shaded?

Answers

Answer:

Step-by-step explanation:

it should be shaded at the 76

What is the equation through the points: (-7, -3), (1, 2)
ASAP please

Answers

The equation of the line passing through the points (-7, -3) and (1, 2) is [tex]y = \frac{5}{8}x + \frac{11}{8}[/tex].

What is the equation of the line?

The formula for equation of line is expressed as;

y = mx + b

Where m is slope and b is y-intercept.

First, we determine the slope of the line.

Given the two points are (-7, -3) and (1, 2)

We can find the slope of the line by using the slope formula:

m = ( y₂ - y₁ ) / ( x₂ - x₁ )

Substituting the values, we get:

m = (2 - (-3)) / (1 - (-7))

m = 5/8

Using the point-slope form, plug in one of the given points and slope m = 5/8 to find the equation of the line.

Let's use the point (-7, -3:

y - y₁ = m(x - x₁)

[tex]y - (-3) = \frac{5}{8}( x - (-7) ) \\\\y + 3 = \frac{5}{8}(x + 7 )\\\\y + 3 = \frac{5}{8}x + \frac{35}{8} \\ \\y = \frac{5}{8}x + \frac{35}{8} - 3\\\\y = \frac{5}{8}x + \frac{11}{8}[/tex]

Therefore, the equation of the line is [tex]y = \frac{5}{8}x + \frac{11}{8}[/tex].

Learn more about  equation of line here: brainly.com/question/2564656

#SPJ1

Solve for w.

w − 5
4
= 2

Answers

The value of the variable w is 56

What are algebraic expressions?

Algebraic expressions are described as expressions that are composed of terms, variables, factors, coefficients and constants.

Also, algebraic expressions are made up of mathematical or arithmetic operations, such as;

AdditionBracketSubtractionParenthesesMultiplicationDivision

From the information given, we have the algebraic equation as;

w - 54 = 2

To determine the value of the variable, we take the steps;

collect the like terms

w = 2 + 54

Now, add the values

w = 56

Learn about algebraic expressions at: https://brainly.com/question/4344214

#SPJ1

Other Questions
. Please calculate the confidence interval based on the conditions below Probability and likelihooda team of scientists is studying the animals at a nature reserve. They capture the animals, mark them so they can identify each animal, and then release them back into the park. The table gives the number of animals theyve identified. Use this information to complete the two tasks that follow. animal total in park number markedelk 5,625 225wolf 928 232cougar 865 173bear 1,940 679mountain goat 328 164deer 350 105moose 215 86part awhat is the probability of the next elk caught in the park being unmarked? write the probability as a fraction, a decimal number, and a percentage. font sizescharacters used: 310 / 15000part bdescribe the likelihood of the next elk caught being unmarked. font sizescharacters used: 58 / 15000part cdescribe a simulation that you can use to model this situation. font sizescharacters used: 374 / 15000part dwhat is the probability of the next wolf caught in the park being unmarked? write the probability as a fraction, a decimal number, and a percentage. font sizescharacters used: 54 / 15000part edescribe the likelihood of the next wolf caught being unmarked. font sizescharacters used: 0 / 15000part fdescribe a simulation that you can use to model this situation. The simulation should be different from the one in part c. font sizescharacters used: 86 / 15000part gin the unit, you found the probability of a compound event by identifying the sample space. However, it is also possible to find the probability of a compound event without finding the sample space. To do this, multiply the probability of the first event by the probability of the second event. For example, the probability of flipping heads twice on a coin is. Using this idea, what is the probability that the next cougar and bear caught will both be unmarked?font sizescharacters used: 0 / 15000part hdescribe the likelihood that the next cougar and bear caught are both unmarked. font sizescharacters used: 0 / 15000part idescribe a simulation that you can use to model this event. font sizescharacters used: 0 / 15000part jusing the method described in part g, what is the probability that the next mountain goat, deer, and moose caught are all unmarked?font sizescharacters used: 0 / 15000part kdescribe the likelihood that the next mountain goat, deer, and moose caught are all unmarked. font sizescharacters used: 0 / 15000part ldescribe a simulation that you can use to model this event. Your simulation should be different from the one in part i. font sizescharacters used: 0 / 15000 On the axes below sketch the graph of y = 4x + 8x +3Label all points of intersection and the turning point in your sketch. The Party invents Newspeak in order to __________.Question 15 options:increase the happiness of Party members and prolesseverely limit the ideas that people can think and talk aboutmake difficult scientific knowledge accessible to all peoplecreate a need for new works of literature the acid-dissociation constant for benzoic acid (c6h5cooh) is 6.3 x 10-5. calculate the equilibrium concentrations of h3o c6h5coo-, and c6h5cooh in the solution if the initial concentration of c6h5cooh is 0.050 m. What is the solution to 7x-4 = 2xO A.4/3O B. -4/4O C. 4/5O D. -4/3 Two balloons are separated by a distance of 25. 5 cm. One balloon is charged with a charge of + 6. 25 nC = + 6. 25 x 10-9 C and the other balloon is charges with a charge of - 3. 5 nC = - 3. 5 x 10-9 C. Calculate the magnitude of Coulombic Force between them. Explain what kind of coulombic force will exist between them (attractive or repulsive)? Talor got a job working for who. What is MOST likely involved in this career?She leads discussions on global health issues.She decides if someone is a good candidate for organ donation.She works in community homeless shelters.She manages people who work at recreation centers. Read the passage from The Tragedy of Romeo and Juliet, Act I. And these, who, often drowned, could never die, / Transparent heretics, be burnt for liars. Use context clues and your knowledge of the Latin prefix trans- to determine the meaning of the word transparent in this context. a- shinyb- glassyc- obviousd- liquid A triangular pane of glass has a height of 32 inches and an area of 256 square inches. What is the length ofthe base of the pane?The length of the base of the pane isinches. Mr. Jones's water supply comes from a well on his property. An underground mine is about to begin operations next door. what two main concerns should mr. Jones have about his water supply?(please answer correctly its due soon 40 points ) When a company is good at performing a particular internal activity, it is said to have: select one: a. a competitive advantage b. a competence c. a resource-based strategy d. benchmark Which process( photosynthesis, cellular respiration, both) do algae preform when incubated in the light? in the dark? Why was it so awful for Gatsby to hear Daisy Buchanan say that she loved them both?NO cheating or plagiarism. You know what consequences youll be dealing with once you break this rule. Itll be major consequences! Lia deposits $100 into a savings account that earns simple interest at a rateof 5%. If she makes no withdrawals, how much interest has Lia's savings account earned after 5 years?A. 45B. 25C. 75D. 15Please explain. Ill give brainly How does Banneker characterize Jefferson in paragraph 3? To assist the sales staff, create a macro on the Pending Orders Status sheet for each of the four salespersons (Markia,Akima, Steve and Juan) that does the following: Filters the list for their name in the Salesperson column Filter to shows Yes in the All Items in Stock column Sorts the results by County in descending order Annan Cob binal SECTION C WRITING COMPOSITION Your composition should be about 150 words long 27th Write a piece for publication in your school magazine about some th are going on in the school that you do not like. SECTION D READING AND COMPREHENSION Read the passage below carefully and answer the questions that follo berin an aerobic exercise, your body needs to war Patricia bought 4 apples and 9 bananas for $12. 70 Jose bought 8 apples and I bananas for $17. 70 at the same grocery store What is the cost of one apple? N2(g)+3H2(g)->2NH3(g), H=-92.40kJ 1. How many grams of H2 are needed to involve 150.9kJ of heat? 2. How many moles of NH3 were produced in the process?